Monday, December 26, 2022

2022/080) Let m and n be positive integers such that gcd(m,n) + lcm (m,n) = m + n. Show that one of the 2 numbers is divisible by the other

Let gcd(m,n) = p.

then m = pq and n = pr for some q and r and gcd(q,r) = 1

gcd(m,n)  = p as we have chosen

lcm(m, n) = pqr as q and r are co-primes

gcd(m,n) + lcm(m,n) = m + n

$=>p + pqr = pq + pr$

$=>1 + qr = q + r$

$=>qr - q -r + 1= 0$

$=>(q-1)(r-1) = 0$

q =1 mean n is divisible by m 

or r =1 meand m is divisible byn

hence proved 

Monday, December 19, 2022

2022/079) Simplify $(\frac{4}{(\sqrt{5}+1)(\sqrt[4]{5}+1)(\sqrt[8]{5}+1)(\sqrt[16]{5}+1)} +1)^{48}$

 as we see that the roots are doubleing in term to term so multiply numeraator and denominator by $(\sqrt[16]{5}-1)$ we get

 $(\frac{4}{(\sqrt{5}+1)(\sqrt[4]{5}+1)(\sqrt[8]{5}+1)(\sqrt[16]{5}+1)} +1)^{48}$

= $(\frac{4*(\sqrt[16]{5}-1)}{(\sqrt{5}+1)(\sqrt[4]{5}+1)(\sqrt[8]{5}+1)(\sqrt[16]{5}+1)(\sqrt[16]{5}-1)}  +1)^{48}$

= $(\frac{4*(\sqrt[16]{5}-1)}{(\sqrt{5}+1)(\sqrt[4]{5}+1)(\sqrt[8]{5}+1)(\sqrt[8]{5}-1)}  +1)^{48}$ using $a^2-b^2$ formula for last 2 terms

= $(\frac{4*(\sqrt[16]{5}-1)}{(\sqrt{5}+1)(\sqrt[4]{5}+1)(\sqrt[4]{5}-1)}  +1)^{48}$ using $a^2-b^2$ formula for last 2 terms-

= $(\frac{4*(\sqrt[16]{5}-1)}{(\sqrt{5}+1)(\sqrt{5}-1)}  +1)^{48}$ using $a^2-b^2$ formula for last 2 terms-

= $(\frac{(4*\sqrt[16]{5}-1)}{4}+1)^{48}$ 

= $(\sqrt[16]{5})^{48} = 5^3 = 125$

2022/078) Given 2 positive nymber a and b with a > b. a-b conatenated with a+b is a 5 digit number c which is prime. find c

 Fir example if  = 5 aand a = 7 then a-b = 2 and a + b = 12. Concatenating we get 212,'


Solution:

because c is prine so $a+b$ is odd so $a-b$ ( as it is $a+b -2b$) is odd. as c is a 5 digit number $a-b$ is 2 digit and$ a+ b$ is 3 digit, mallest $a-b$ is 11 and $a+b$ is odd staring from 101 putting the values we get $a+b = 113$ and $a -b = 11$ and hence c = 11113.

Sunday, December 11, 2022

2022/077) If $a^2+b^2+c^2 = 1$ find the range of $ab + bc+ca$

We need to find the mininal and maximal of $ab+bc+ca$

we have $(a+b+c)^2 = a^2 + b^2 + c^2 + 2(ab+ bc+ ca)$

so $ a^2 + b^2 + c^2 + 2(ab+ bc+ ca) > = 0$

puttig $a^2+b^2+c^2 = 1$ we get

$ 1 + 2(ab+ bc+ ca) > = 0$

or $(ab+bc+ca) >=  - \frac{1}{2}$

Further to find tthe maximum we have $(a-b)^2 + (b-c)^2 + (c-a)^2 = 2(a^2 +b^2+c^2 - ab - bc - ca)$

or $2(a^2 +b^2+c^2 - ab - bc - ca) >= 0$

or $ab+bc+ca <= a^2+b^ + c^2$

or $ab+bc+ca <= 1$

so we have $ab+bc + ca \in [-.5 .. 1]$



 

Saturday, December 10, 2022

2022/076) Find prime p such that $16p+1$ is a cube

We have $16p + 1$ is a cube say $x^3$

So $16p= x^3-1 = (x-1)(x^2+ x + 1)$

As $x^2+x+1 = x(x+1) +1$ is odd we must have

$16 | x-1$

So $x = 16k+1$ for some integer k

so $p = k(x^2 + x + 1)$

the above cannot be prime unless k =1 and this gives x = 17 and p = 307

We need to see if  307 is prime and it is so

Hence the only solution is p = 307 (giving $16 * 307 +1  = 17^3$

 


 


Friday, December 2, 2022

2022/075) Given the two number patterns: 1,5,9,13,17... and 1,4,7,10,13... How do I find their 13th common number?

 in the 2 patterns the 1st number is same.

now 1st pattern has common diffrerence 4 and 2nd one 3

the minimum common number difference shall be LCM(4,3) or 12

so 13th common number shall be 1 * 12 * (13–1) = 145

2022/074) if a and b are roots of $x^2-3x+1=0$ then find the value of $(a^a + b^b)(a^b+ b^a)$

 We are given f a and b are roots of $x^2-3x+1=0$

so $a + b = 3\cdots(1)$
$ab=1\cdots(2)$2

now $ $(a^a + b^b)(a^b+ b^a)$$

= $a^{a+b} + (ab)^a + (ba)^b  b^(b+a)$

= $a^3 + 1 + 1 + b^3$ putting the value of a+b and ab from (1) and (2)

=$a^3 + b^3 + 2$

= $(a+b)^3 - 3ab(ab+b) + 2$ using formula for $a^3+b^3$

= $3^3 - 3 . 1 . 3 + 2 = 20$ putting values from (1) ansd (2) 


Sunday, November 27, 2022

2022/073) Maximize $x \sqrt{1-y^2} + \sqrt{1-x^2}$

 In the above expression x  not dependent on y term so let us maximize $\sqrt{1-y^2}$ and this is {maxium at y= 0 and the value is 1.

Putting the same we get $x  + \sqrt{1-x^2}$

As we have $\sqrt{1-x^2}$ we can put $x=\sin\,t$ ( $0 \le t \le \frac{\pi}{4}$ to get

$x  + \sqrt{1-x^2}= \sin\, t + \cos \, t $

We need to maximize $\sin\, t + \cos \, t $ so let us get in form $r \cos\, w \sin \, t + r \sin\, w \cos  \, t$

which is $r \sin ( t+w)$ and maximum value is |sqrt{2}$

So let $r\cos\, t = 1$ and $r \sin\, t = 1$

By squaring both and adding 

$r^2 = 2$ or $r=\sqrt{2}$

Hence largest valus is $\sqrt{2}$

 


 

Saturday, November 26, 2022

2022/072) find positive integers x and y such that $\frac{1}{y} - \frac{1}{y+2} = \frac{1}{3 * 2^x}$

 We have multipying by $x(y+2)$ 

$\frac{2}{y(y+2)}= \frac{1}{3 * 2^x}$

or $ 3 * 2^{x+1} = y(y+2)$

let z = x+1 so

 $ 3 * 2^{x+1} = y(y+2)$

LHS is even so y is even because if y is odd then y(y+2) is odd

3 is a factor of y or y + 2

let us atke the 2 cases one by one

case 1 :3 is a factor of y

so $y = 3 * 2^a$ and $y+2 = 2^b$ for some inetger a and b 

$3 *2^a + 2 = 2^b$

or $2(3 * 2^{a-1} + 1) = 2^{b+1}$

as RHS is even so $3*2^{a-1}$ is odd or  a = 2

so y = $3 *2^a = 6$ and $y+2 =  8$ from this $3 * 2^{x +1} = 48 = 3 * 2^4$ ot x = 3

case 2 :3 is a factor of y +2

so $y+ 1 = 3 * 2^a$ and $y-2 = 2^b$ for some inetger a and b 

$3 *2^a - 2 = 2^b$

or $2(3 * 2^{a-1}-1 = 2^{b+1}$

as RHS is even so $3*2^{a-1}$ is odd or  a = 2

so y = $3 *2^a = 6$ and $y -2 =  4$ from this $3 * 2^{x +1} = 24 = 3 * 2^3$ ot x = 2  

So 2 solutions are (3,6) and (2,4)

  


Sunday, November 13, 2022

2022/071) Find integers a,b,c such that $a^2-b^2 = 24$ and $b^2-c^2= 16$

We ahve $a^2-b^2 = 24\cdots(1)$

$b^2-c^2 = 16\cdots(2)$ 

Because $a^2-b^2 = 24$ so both a and b are odd or both ae even,

So we have a = b + 2n for some n

To put an upper bound on be a = b+ 2 beause a is at least 2 more than b 

So we have $a^2-b^2 = (b+2)^2 - b^2  = 24$

Or $2b + 4 = 24$ or $b= 5$

For the lower bound $b^2 =17$ so $b ge 5$

So we get b = 5 and putting the values 

So we get $a= 7, b = 5, c= 3$ 

2022/070) 7 dices are rolled. What s the probabilty that the sum of top faces is 10 ?

 For eah dice there are 6 outcomes

for 7 dices there are $6^7$ outcomes

so there are $5^7$ out comes

10 can come in followin ways

6 ones and a 4 this can come in 7 ways(4 at any of 7 places)

5 ones one 2 and one 3 this can come in 7 * 6 = 42 ways( 2 at any of  7 places and and then 3 in any of the next 6 paces)

4 ones and 3 twos = $7 \choose 3$ = 35 places

so total number of places = 7 + 42 + 35 = 84.

so probability = $\frac{84}{6^7}$


  

Monday, October 31, 2022

2022/069) find the pythegorean triplet whose one number 10x+ y and largest number 10y + x

let the 3rd number be t

so $t^2 + (10x+y)^2 = (10y+x)^21$

or $t^2 = (10y+x)^2 - (10y+x)^2 = 99(y^2 - x^2) = 99(y-x)(y+x)= 11 . 3^2(y+x)(y-x)$

y+x is less than 18 and y-x is less than 10 

for the RHS to be a perfect square we must have y +x = 11 and y-x is to be perfect squiare and odd

because 11 cannot be a factor of y-x and if y +x is odd then y-x has to be odd because y-x = y + x - 2x

so y+ x = 11 and y-x = 1 giving y = 6 and x = 5

hence $t^2 = 99 * 11 * 1$ or t = 33

so triple is (33,56,65)

 

Saturday, October 22, 2022

2022/068) Find the smallest positive number n such that $3n^3-2019$ is divisible by 2016

 $3n^2-2019$ is divisible by 2016

Both are mutiple of 3 so $n^3-667$ is divisible by 672

Or  $n^3-667+672$ that is $n^3-1$ is divisible by 672

Now $n^3-1= (n-1)(n^2+n+1)$ and 672 = 32 * 21 (product of even and odd)

As $n^2+n+1 = n(n+1) +1 $ is always odd so we have (n-1) multiple of 32.

Futher we must have $n^3 \equiv 1 \pmod 3$ and  $n^3 \equiv 1 \pmod 7$

For $n^3 \equiv 1 \pmod 3$ we take mod 3 and get 0 for 0 1 or 1 and 2 for 2 so n must be 1 mod 3

Or f n-1 must be mutiple of 3

So $n \equiv 1 \pmod {96}$

Futher we must have $n^3 \equiv 1 \pmod 7$ and  $n^3 \equiv 1 \pmod 7$

for $n^3 \equiv 1 \pmod 7$ we take mod 7 and get 0 for 0,1 for 1,2,4,  and 6 for 3,5,6

So we must have n = 1/2/4 mod 7

n = 96k + 1

k =1 gives n = 97 , mod 7 = 6 so not a sulution

k = 2 gives n = 193, mod 7 = 4 so is a solution

So ans smallest n = 193


 


Sunday, October 16, 2022

2022/067) Find the no of solutions of equation $\frac{1}{m} + \frac{1}{n} = \frac{1}{143}$ where m , n are distinct positive inetegers

 We have $\frac{1}{m} + \frac{1}{n} = \frac{1}{143}$

or $143n + 143m = mn$

or mn - 143m - 143n = 0

this if of the form $m(n-143) - 143n = 0$

To solve these type of probem add and $143^2$ to both sdes to get

 $m(n-143) - 143(n- 143)= 143^2$

or $(m-143)(n-143) = 143^2= 11^2 * 13^2$

we get the following pairs as solutions (n-143,n-143) = (1,20449), (11, 1859), (13,1573), (121,169),(143,143), (169,121), (1573,13), (1859,11),(20449.1)

or (n,m) = (144, 20592), (11,2002),(13,1716), (264, 312), (286,286), (312,264),(1716,13),(2002,11),(20592.144)

there are 9 pairs out of which 8 are distinct  

Saturday, October 15, 2022

2022/066) Let a,b be real numbers (b≠0) and consider the infinite arithmetic sequence a,a+b,a+2b,⋯. How do I show that this sequence contains an infinite geometric subsequence if$\frac{a}{b}$ is rational?

Let $\frac{a}{b}$ be rational and so let it be $\frac{n}{m}$ where n and m are integers

so $\frac{a}{b}= \frac{n}{m}$ or $b= \frac{ma}{n}$

now let $t_n$ the the nth term

so $t_(n+1) = a + ma = a(1+m)$

to get $a(1+m)^2$ we need to add $a(m^2 + 2m)$ or $am(m+2)$ as $nb = ma$ so $nb(m+2) = am(m+2)$ and so we get this term as well

to get $a(1+m)^k$ we need to add $a((1+m)^k -1)$ or $amf(m)$ as $nb = ma$ so $nbf(m) = amf(m)$ and so we get this term as well

so we get the terms in GP

2022/065) Find the number of 3 digit numbers whose GCD with 36 is 2

 Let n ba me number.

now $36 = 2 ^ 2 * 3^2$

$gcd(n, 36) = 2$  means 2 is a facttor bf n but is is not a factor of n. and 3 is not a factor of it

So nwe take the multiple of 2 and remove from this mutipls of 4 and multiples of 6.

because we need to remove multiple of 3 so we need to remove multiple of 6 (as they are multiple of 2) 

To find the number of numbers 

the number of numbers which are mutilple of 2 = $\lfloor\frac{999}{2}\rfloor - \lfloor{99}{2}\rfloor = 449 - 49 = 500$

the number of numbers which are mutilple of 4 = $\lfloor\frac{999}{4}\rfloor - \lfloor{99}{4}\rfloor = 249 - 24 = 225$

the number of numbers which are mutilple of  6 = $\lfloor\frac{999}{6}\rfloor - \lfloor{99}{6}\rfloor 166  - 16 = 150$

multiple of 4 and 6 both contain multiples of 12(LCM of 4 and 6).

mutiples of 12 will be in both the lists that is multiple of 4 and 6.

the number of numbers which are mutilple of  12= $\lfloor\frac{999}{12}\rfloor - \lfloor{99}{12}\rfloor 83  - 8 = 75$

so number of mutiples of 4 and 6 are 225 + 150 -75 = 300

so number of numbers whose GCD with 36 =2 is 450 - 300 = 150





Thursday, September 22, 2022

2022/064) How to solve the Diophantine equation ab+a+b+82=z2, where (a,b,z) are positive integers & g.c.d(a,b)=1?

We have $ab + a + b + 82 = z^2$

or $ab + a + b + 1 + 81 = z^2$ we do so that we can facrot LHS

or $(a+1)(b+1) = z^2 - 81 = (z-9)(z+9)$ 

we can choose a +  1 = z - 9 and b+1 = z + 9 (this shall not give all solutions but some 

so a = z - 10 and b = z + 8

they have to be co-prime so both are odd say z = 2 m + 1

so a = 2m - 9 and b = 2m + 9

for these to be co-prime there should not a factor of 18 (that is the difference)

so 2m - 9 shall not have a factor 3 or m must not have a factor 3

so a = 2m - 9 and b = 3m + 9 m > 5 and of the forn $3k \pm 1$

 


2022/063) How can I find all positive integer values of N such that N(N−101) is a square of a positive integer

 Now let us take GCD(N,N-101)

GCD(N,N=101) = GCD(N,101)

There are 2 cases either N is multiple of 101 or not

If N is multiple of 101 say N= 101m

So N-101 = 101(m-1) so $N(N-101) = 101^2m(m-1)$ which is not a perfect square

So N is not a multiple of 101 so GCD(N,N-101) = 1 so N and N-101 both are perfect squares

Say $N= x^2$ and $N-101$ = y^2

$x^2-y^2 = 101$ or $(x+y)(x-y) = 101$ 

So x + y = 101 and x-y =1 solving these we get x = 51 and y = 50

So $N = x^2 = 51^ = 2601$

Thursday, September 8, 2022

2022/062) Prove that every number in the sequence 16,1156,111556,⋯ is a perfect square.

We have for $n^{th}$ number it is n 1s followed by n-1 5's and ending with 6

of it is n 1's follwed by n 5's and add 1

n '1s $\frac{1}{9}(10^n-1)$

n 5's is   $\frac{5}{9}(10^n-1)$

so the number is  $\frac{1}{9}(10^n-1) * 10^n + \frac{5}{9}(10^n-1) + 1$

$= \frac{1}{9}(10^{2n}-10^n +  5* (10^n-1) + 9)$

$= \frac{1}{9}(10^{2n} +  4* 10^n + 4)$

$=\frac{1}{9} (10^n + 2)^2$

$=(\frac{1}{3}(10^n+2))^2$

which is  peffect square 




Sunday, September 4, 2022

2022/061) What are the values of n for which $2^4+2^7+2^n$ is a perfect square?

 We have $2^4+2^7 + 2^n = 2^n + 144 = m^2$ where m is positive 

or $2^n = m^2 - 144 = (m+12)(m-12)$

as 2 is a prime so both m+ 12 and m-12 powers of 2

now difference (m+12)- (m-12) = 24

so powers of must have a difference 24

as $2^5 = 32$ so $2^6-2^k \ge 2^6 - 2^5 > 32$ (for any k less of equals 5)

so we need to check for candidates 2,4,8,16,32 and get 8 and 32.

this gives m = 20 and putting m = 20 we get n= 8

Friday, September 2, 2022

2022/060) If $x^2 - 3x +1 = 0$ and $x^{16} - k x^8 + 1 = 0$ then find k

Let $y^2- my + 1 = 0\cdots(1)$

so $y^2 = my -1$

or squaring both sides $y^4 = m^2y^2 - 2my + 1 = m^2y^2 - 2(y^2) + 1= (m^2-2)y^2$

or $y^4 = (m^2-2)y^2 + 1\cdots(2)$

putting y =x and m = 3 we get

$x^2 - 3x + 1$

and from (2) we have $x^4 - 7x^2 +1 = 0\cdots(3)$ 

putting $y = x^2$ and m = 7 in (1) we get (3) and from(2)  we get 

$x^8 - 47x^4 +1 = 9\cdots(4)$

  putting $y = x^4$ and m = 47 in (1) we get (4) and from(2)  we get  

2022/059) If $4x=a^2+3$ where (a,x) are positive integers & x is not a multiple of 3, then It's only possible when x are certain positive integers of the form of 6y+1. Prove this?

 Because 4x is even so we have a is odd 

So let a = 2n + 1

$4x= (2n+1)^3 + 3 = 4n^2 + 4n + 4$

or $x= n^2 + n + 1 = n(n+1) + 1$

now n can be 0/1/2 mod 3

if n is one of the form $(3k, 3k+1, 3k-1$

If n is of the form 3k then $x-1 = 3k(3k+1)$ divisible by 6 so x of the orm 6m + 1

if n is of the form 3k + 1 then $x-1 = (3k+ 1)(3k+2) = 9k^2 + 9k^2 + 2 = 9k(k+1) + 2$ remainder 2 when devided by 6 

so x = 6m + 3 but is divisible by 3 so not admissible

if n is 3k -1 then 

$x-1 = (3k-1)(3k)$ of the form 6m or x+ 1of the form 6m + 1

So it is of the form 6m + 1(or 6y + 1)  and hence proved   


Saturday, August 20, 2022

2022/058) Given $\frac{1}{x} + \frac{1}{y} = \frac{1}{x+y}$ find $\frac{x}{y} + \frac{y}{x} $

Multiply both sides by x+ y to get

$\frac{x+y}{x} + \frac{x+y}{y} = 1$

or $ (1+ \frac{y}{x}) + (\frac{x}{y} + 1)  = 1$

or   $ \frac{y}{x} +\frac{x}{y} + 2  = 1$

or $\frac{y}{x} + \frac{x}{y}   = -1$

2022/057) Given $(a-5)^2 + (b-7)^2 = 4$ find the minimum of $(a+7)^2 + (b+2)^2$

$(a-5)^2 + (b-7)^2 = 4$ is a circle whose centre is $(5,7)$ and radius is 2

We need to find minimum of $(a+7)^2 + (b+2)^2$ that is distance of the point A with coordinates a,b  fom (-7,-2).

This is minimum when the point lies on the straight line from (-7,2) , (5,7) . 

this distane from (-7,2) to (5,7) = $\sqrt{(5+7)^2 + (7-2)^2} = 13$

the distance of a from (5,7) is 2

so distance of point from (-7,2)is 13 - 2 = 11

so minmum of  $(a+7)^2 + (b+2)^2$ is 121

  

Thursday, August 4, 2022

2022/056) What is the value of a, b, c If $2^a+4^b+8^c=328$ and a, b, c are natural numbers?

 Let us express 328 as sum of power of 2 (as 2,4,8 all are power of 2)

$328 = 256 + 64 + 8 = 2^8 + 2^ 6 + 2^3$

Only $2^6$ or $2^3$ are power of 8

So let us consider these 2 cases

1) $2^6 = 8^2$ and 8 is not power of 4 so $8 = 2^3$ and $256=4^4$ giving a = 3, b = 4, c= 1

2) $2^3 = 8^1$ and this gives 2 cases 

        case 1 $4^4 = 256, 2^6 = 64$ giving a = 6, b = 4, c = 1

        case 2  $2^8 = 256, 4^3 = 64$ giving a = 8, b = 3, c = 1

The above 3 are solutions 

Saturday, July 23, 2022

2022/055) 2020 distict numbers are in a circle. Show that we can always choose 4 consecutive numbers such that sum of outer 2 numbers is greater than tsom of 2 inner numbers

The 2020 numbers are in a circle. Let the numbers be $a_1$ to $a_{2020}$ now startting wth $a_k$ we have 4 number consecutive $a_k,a_{k+1} ,  a_{k+2}$, $a_{k+3}$ for k from 1 to 2017 and other  4 consecutive  number $a_{2018},a_{2019}, a_{2020}.a_1$ so on

now let us compute the value sum of outer numbers - sum of innter numbers calling $S_n$

$S_{1} = a_{1} - a_{2} - a_{3} + a_{4}$

$S_{2} = a_{2} - a_{3} - a_{4} + a_{5}$

...

...

$S_{2020} = a_{2000} - a_{1} - a{2} + a_3$

If we add all of these then we have

$\sum_{n=1}^{2020}S_{n} = 0$

For the sum of 2020 numbers be zero either all are zero (that means some numbers are same which is not true ) or at least one number is positive say $S_{k}$ then $a_{k}$ is the starting point

 


Saturday, July 16, 2022

2022/054) Number $3^{32}-1$ has exactly two divisors greater than 75 and less than 85. What is their product?

we have $3^{32}-1 =  (3^{16}+ 1)(3^{16}-1) =   (3^{16}+ 1)(3^{8}+1) (3^{8}- 1)$

$ = (3^{16}+1)(3^8+1)(3^4+1)(3^4-1)$ 

out of the above $3^4+1 = 82$ and %3^4-1=80$ are between 75 and 85 and product is 80 * 82 = 6560

2022/053) If $x^2-x=1$ Symplify $\frac{x^5+8}{x+1}$

 We have $x^2 = 1-x\cdots(1)$

multiply both by x to get

$x^3 = x -x^2 = x - (1-x) = 2x -1\cdots(2)$

 $x^5 = x^3 .x x^2 = (2x-1)(1-x) = -2x^2 +3x -1 = -2(1-x) + 3x - 1 = 5x-3$ (using (1))

or $x^5 + 8 = 5x + 5$

or  $\frac{x^5+8}{x+1} = 5$

2022/052) Simplify expression $(\frac{1}{2^3-1} + \frac{1}{2}) (\frac{1}{3^3-1} + \frac{1}{2}) \cdots (\frac{1}{100^3-1} + \frac{1}{2})$

We have let $t_n =   \frac{1}{t^3-1} + \frac{1}{2}$

$= \frac{1}{2}\frac{t^3+1}{t^3-1}$

=$\frac{1}{2}\frac{(t+1)(t^2-t+1)}{(t-1)(t^2+t+1)}$

Now given value = $\prod_{n=2}^{100} \frac{1}{2}\frac{(t+1)(t^2-t+1)}{(t-1)(t^2+t+1)}$

$= \frac{1}{2^{99}} \prod_{n=2}^{100} \frac{(t+1)}{(t-1)}\frac{(t^2-t+1)}{(t^2+t+1)}$

now let us compute the 2 products 

$ \prod_{n=2}^{100} \frac{(t+1)}{(t-1)}= \frac{3}{1}.\frac{4}{2}.\frac{5}{3}\cdots \frac{98}{96}. \frac{99}{97}. \frac{100}{98}. \frac{101}{99}$

In the above product the numerator of $n^{th}$ term is same as denoniator of $n+2^{nd}$ term giving

$ \prod_{n=2}^{100} \frac{(t+1)}{(t-1)} = \frac{100 * 101}{1 * 2}\cdots(1)$

let us compute 2nd product

$  \prod_{n=2}^{100} \frac{(t^2-t+1)}{(t^2+t+1)}= \frac{3}{7} ( \frac{ 7}{13} \cdots \frac{9901}{10101}$

In the above product the numerator of $n^{th}$ term is same as denoniator of $n-1^{st}$ term giving

$  \prod_{n=2}^{100} \frac{(t^2-t+1)}{(t^2+t+1)}= \frac{3}{10101}\cdots(2)$

using (1) and (2) we get the required result

$= \frac{1}{2^{99}}\frac{3}{10101} * 5050= \frac{2525}{2^{98} * 3367}$

Sunday, July 10, 2022

2022/051) Show tthat there are infnite solutions to $x^3+y^4= z^{31}$

We know that 2^m * 2 is a power of 2

so if we chose $x^3 = y^4 = 2^t$ for some t then we shall get power of 2

so let $x = 2^{4m}$ and $y = 2^{3m}$

so $x^3 + y^ 4= 2^{12m} + 2^{12m} = 2^{12m+1}$

 Now there is only one 1ssure remanins that is 12m + 1 has to be multiple of 31

that is not a problem as gcd(31,12) is 1 and we shall use exteneded euclidean algoirithm to find the same

now 31 =  2 * 12 + 7 we take the highest multiple of 12 and leave the remainder

or $7 = 31 - 2 * 12\cdots(1)$

12 = 7 * 1 + 5

or $5 = 12 - 7 * 1 = 12 - (31 - 2 * 12)$ using (1)

or  $5 = 12 * 3 - 31\cdots(2)$

now $ 7 = 5 + 2$

or $2 =  7- 5 = (31- 2 * 12) - (12 * 3 -31) = 2 * 31 - 5 * 12$ (uinsg (1) and (2)

or $2 = 2 * 31 - 5 * 12$

now $5 = 2 * 2 + 1$

or $1= 5 - 2 * 2$

or $1 = (12 * 3 - 31) - 2 * ( 2 *31 - 5 *12) = 13 * 12 - 5 * 31$

so if m is 13 or any number 31n + 13 we get 12m+ 1 is a multiple of 31

m = 31n + 13 gives 12m +1 = 12(31n + 13) = 31(12n + 5)

so $x = 2^{4(31n + 13)}$, $y = 2^{3(31n + 13)}$ and $z = 2^{12n+5}$ is parametric solution and hence infinite solutions.



2022/050) Find n such that $n! = 3!5!7!$

 clearly n > 7.

now if t ia factorial then 3!5! must be product of sum k sucesssive numbers from 8

3!5! = 6 * 120 = 720 = 8 * 90 = 8 * 9 * 10

so n! = 10 * 9 * 8 * 7! = 10! 

or n = 10


Saturday, July 9, 2022

2022/049) Prove that if a number is sum of 3 squares then square of the number is sum of 3 squares

 Let n be sum of 3 squares so

$n= a^2 + b ^2 + c^2$

we need to show that $n^2$ is sum of 3 squares

from given condition

$n^2 = (a^2+b^2+c^2)^2 = a^4 + b^4 + c^4 + 2 a^2b^2 +2 b^2 c^2 + 2 c^2 a^2$

$=  a^4 + b^4 + c^4 + 2 a^2b^2 -2 b^2 c^2 - 2 c^2 a^2 + 4 b^2c^2 + 4 c^2 a^2$

$=(a^2+ b^2 -c^2)^2 + (2bc)^2 + (2ca)^2$

sum of 3 sqaures and hence proved 

 

2022/048) Given $\frac{a^2}{b+c} = \frac{b^2}{c+a} = \frac{c^2}{a+b} = 1$ Find $\frac{1}{a+1} + \frac{1}{b+1} + \frac{1}{c+1}$

 We are given

$\frac{a^2}{b+c} = 1$

or $a^2 = b + c$

or $a^2 + a = a + b + c$

or $a(a+1) = a + b + c$

or $\frac{1}{a+1} = \frac{ a}{a + b+ c}\cdots(1)$

similarly  $\frac{1}{b+1} = \frac{a}{a + b+ c}\cdots(2)$

and  $\frac{1}{c+1} = \frac{a}{a + b+ c}\cdots(3)$

adding all 3 above we get  $\frac{1}{a+1} + \frac{1}{b+1} + \frac{1}{c+1} = \frac{a+b+c}{a+b+c} = 1$


Thursday, June 30, 2022

2022/047) Solve in integers $y^2 - 2001 = n!$

 We have 2001 is divisible by 3 and not 9. And if n Is above 5 then n! is divisible by 9. So $2001 + n!$ is divisible by 3 but not 9. so it cannot be a perfect square. so checking from 0 to 5 we get y = 45 and n =4


Sunday, June 12, 2022

2022/046) Find all n such that $7 | 2^n-1 $ and show that there is no positive n such that $7 | 2^n + 1$ (IMO 1964 problem 1)

because 7 is a prime so as per Fermats Little theoren 

$ 7 | 2^6-1$

now as $2^6$ leaves remainder 1 after dividing by 7 so it may be taht for some factor a of 6 $7 | 2^a-1$

we need to check for 1,2,3

so see $2^1-1 = 1$ $2^2-1 = 3$ and $2^3 - 1 = 7$ out f these 3 3 satisfies

as so k = 3m for all m satisfies.

Further $2^1+1 = 3$ $2^2+1 = 5$ and $2^3 +1 = 9$ out f these 3 none is divsible by 7 so there is no n such that $ 7 | 2^n+1$ 

proved 


2022/045) Find x given $(2+\sqrt{3})^x + (2-\sqrt{3})^x = 4$

 we see that $(2+\sqrt{3})(2-\sqrt{3}) = 4 - 1 = 1$

so $\frac{1}{(2+\sqrt{3}} = 2 - \sqrt{3}$

Let $(2+\sqrt{3})^x = y$ so $(2-\sqrt{3})^x = \frac{1}{y}$

so $y +\frac{1}{y} = 4$

or $y^2 - 4y + 1 = 9$

ot $y = \frac{4 \pm \sqrt{16-4}}{2}$

or $y= 2 \pm \sqrt{3}$

If we take $y = 2 + \sqrt{3}$ then $(2+\sqrt{3})^x = 2 + \sqrt{3}$ or $x= 1$

 Taking $y = 2 - \sqrt{3}$ then $(2+\sqrt{3})^x = 2 - \sqrt{3}$ or $x= -1$

so $x \in \{ 1, -1\}$


Tuesday, June 7, 2022

2022/044) If p, q, r, s are consecutive terms of an arithmetic sequence with common difference n how do you prove that $pqrs + n^4$ is a perfect square?

 Without loss of generality set n = 2x ( x need noy be integer so we can choose  

p = m - 3x, q = m - x, r = m + x, s = m + 3x

now $pqrs+n^2 = (m-3x)(m-x)(m+x)(m+3x)  + 16x^4$

$= (m- 3x)(m+ 3x)(m-x)(m+x) + 16x^4$ 

$= (m^2-9x^2)(m^2- x^2) + 16x^4$

$= m^4 - 10x^2m^2  + 9x^4 + 16x^4$

$=m^2 - 10x^2m + 25x^4 = (m^2 -5x^2)^2$ so it is a perfect square 

as it is integer square root is also integer 


Monday, June 6, 2022

2022/043) Sum of 13 numbers is 1000 what is the minimum LCM

The sum of 23 numbers is 1000 so the numbers can be 12 numbers 77 and 76, so the LCM>=77.

 the LCM is lower if 13th number is a factor of 12 numbers (which are same)

let us look at factors of 1000. and one factor is minumum 13.

we have 50 * 20. 7 numbers  can be 100 and 6 can be 50 giving LCM = 100

40 * 25, 12 numbers 80 and one number 40 giving LCM = 80

25 * 40 12 numbers can be 75 but 13th number is 100 LCM = 300

20 * 50  12 numbers 80 and one number 40 giving LCM = 80

So lowest LCM = 80 (12 numbers 80 and one number 40) 

Sunday, May 29, 2022

2022/042) Solve for x $(1+ \frac{1}{x})^{x+1} = (1+ \frac{1}{n})^n$

 we have

$(1+ \frac{1}{x})^{x+1} = (\frac{x+1}{x})^{x+1}$

$=(\frac{x}{x+1})^{-(x+1)}$

$=(1- \frac{1}{x+1})^{-(x+1)}$

$= (1+ \frac{1}{-(x+1)})^{-(x+1)}$

comparing with the RHS we get -(x+1) = n  or x = -(n+1) 

Sunday, May 15, 2022

2022/041) Find the digit X such that 9986860883748524X5070273447265625 equals $1995^{10}$

Because 1995 is divisible by 3 so all the powers $ ≥2 $ of 1995 shall be divisible by 9.

So sum of digits of the result must be divisible by 9.

We get sum of digits of given number = 160 + X.

Smallest multiple of 9 above 160 is 162 which gives X= 2 and next multiple of 9 gives X = 11.

so X = 2 


 

2022/040) If $x = 3 + \sqrt[3]3 + \sqrt[3]{3^2}$ then show that $x^3 - 9x^2+ 18x -12 = 0$

 From the given condition we have

$(x-3) = \sqrt[3]3 + \sqrt[3]{3^2}$

squaring both sides

$(x-3)^3 = (\sqrt[3]3)^3 + ( \sqrt[3]{3^2})^2 + 3  \sqrt[3]{3} \sqrt[3]{3^2}(  \sqrt[3]{3} +  \sqrt[3]{3^2})$ using $(a+b)^3 = a ^3 + b^3 + 3ab(a+)$

or $(x-3)^3 = 3 + 9 + 9 ( x-3)$ using $(  \sqrt[3]{3^2} +  \sqrt[3]{3^2} = x- 3$

or $(x^3 - 9x^2 + 27 x - 27 = 12 + 9x - 27 = 9x + 12$

or $x^3 - 9x^2+ 18x -12 = 0$

2022/039) Find x integer such that $\frac{8^x - 2^x }{6^x - 3^x} = 2$

 We have numerator = $8^x - 2^x = (2^3)^x - 2^x = (2^(3x) - 2^x = 2^x(2^{2x} - 1)= 2^x(2^x + 1)(2^x-1)$

Denominator = $6^x - 2^x = 3^x(2^x-1)$

So $\frac{8^x - 2^x }{6^x - 3^x} = \frac{2^x(2^x+1)}{3^x} = 2$

Or $2^x(2^x+1) = 2 * 3^x$

x cannot be zero as in the original question x = 0 is not possible

So we must ahve $2^x = 2$ and $2^x+ 1 = 3^x$ by equating even on both sides and odd on both sides

1st equuation gives x = 1 and it satisfies 2nd equation as well.

so Solution x = 1 


Saturday, May 14, 2022

2022/038) The sides of a right angled triangle are in arithmetic progression. If the triangle has area 24, find the length of smallest sides

Let the sides be a -b , a , a+b

as the tringle is right angled so $(a-b)^2 + a^2 = (a+b)^2$

or $a^2-2ab + b^2 + a ^2 = a^2 + 2ab + b^2$

or   $a^2= 4ab$

or $a= 4b$

as the triangls is right angled so aea = $\frac{1}{2} a(a-b) =  \frac{1}{2} 4b(4b-b) = 6b^2 = 24$ or b= 2

the smallest sde = 3b = 6. 

2022/037) If a, b, c are in H.P., how do you prove that $\frac{b+c-a}{a}$, $\frac{c+a-b}{b}$, $\frac{a+b-c}{c}$ are in A.P.​?

 a,b,c are in HP so $\frac{1}{a}$, \frac{1}{b}$, $\frac{1}{c}$ are in AP

or $\frac{1}{b} - \frac{1}{a} = \frac{1}{c} - \frac{1}{b}$

multiply both LHS and RHS by a + b + c we get

$\frac{a+b+c}{b} - \frac{a+b+c}{a} = \frac{a+b+c}{c} - \frac{a+b+c}{b}$

Or 

$(\frac{a+b+c}{b} - 2) - (\frac{a+b+c}{a}-2)  = (\frac{a+b+c}{c} -2)  - (\frac{a+b+c}{b} -2) $

or $(\frac{a+c-b }{b}) - (\frac{b+c-a }{a})  = (\frac{a+b-c }{c})  - (\frac{c + a - b}{b} $)

Hence  $\frac{b+c-a}{a}$, $\frac{c+a-b}{b}$, $\frac{a+b-c}{c}$ are in A.P

Saturday, May 7, 2022

2022/036) Let $t_n= \sin^n \theta + \cos^n \theta$ show that $6t_{10} − 15t_8 + 10t_6 = 1$ .

Let us put $p = \sin ^2 \theta \cos^2 \theta$  

We have $t_2 = \sin ^2 \theta + \cos^2 \theta = 1 \cdots(1)$

and $t_4 = (\sin^4x + \cos^4 x) = (\sin ^2 x + \cos^2 x) - 2 \sin ^2 x +\cos^2 x = 1 - 2p$ 

Now we an sert 

$t_n t_2 = ( \sin ^n \theta + \cos^n \theta)(\sin ^2 \theta + \cos^2 \theta) = \sin  ^{(n+2)}\theta +  \cos^{(n+2)} + \sin ^n \theta \cos^2 \theta +    \sin ^2 \theta \cos^n \theta$

or $t_n = t_{(n+2)} + t_{(n-2)}p$

or $t_(n+2) = t_n - t(n-2)p$

Using this we have

$t_6 = t_4 - t_2p = t_4 - p = 1- 3p$

 $t_8 = t_6 - t_4p = 1-3p -  (1-2p ) p^2 = 1- 4p + 2p^2$

$t_{10} = t_8 - t_6p  = (1-4p +2p^2) - (1-3p)p = 1 - 5p + 5p^2$

so $6t_{10} − 15t_8 + 10t_6 = 6(1-5p + 5p^2 ) - 15(1- 4p + 2p^2 ) + 10(1-3p)  = 1 $

  


Tuesday, May 3, 2022

2022/035) Find the smallest prime factor of $2019^8 + 1$

let the prime be p

we have $2019^8 \equiv -1 \pmod p$ 

or $2019^{16} \equiv 1 \pmod p$

so $2019^{16k} \equiv 1 \pmod p$

as per Formats Little theorem p shall be of the form 16k + 1.

any number less than 16 is not possible because  $2019^8 \equiv -1 \pmod p$ and $16=2^4$ and $8=2^3$ so any factor of 16 shall not give a power =1.

so p = 16 *2 + 1 = 33 (not a prime) or 16 * 3 + 1= 49( not a prime) or 16 * 4 + 1 = 65( not a prime) or 16 * 5 + 1 = 81 (not a prime) or 16 *6  + 1 = 97.

so we check for 97 

now $2019 \equiv - 18 \pmod {97}$ 

so $2019^2 \equiv 324 \pmod {97}$

or   $2019^2 \equiv 33  \pmod {97}$

so $2019^4 \equiv 33^2  \pmod {97}$

or $2019^4 \equiv 22   \pmod {97}$

or $2019^8 \equiv 484  \pmod {97}$

or $2019^8 \equiv -1  \pmod {97}$

or or $2019^8 + 1\equiv 0   \pmod {97}$

so smallest prime factor = 97 

Sunday, May 1, 2022

2022/034) How many real roots does $x^ 4 + 12x − 5$ have?

 let $f(x) = x^4+ 12x--5$

As there change of sign once in f(x) so there is one positve real toot as per descartes' rule of signs

$f(- x) = x^4- 12x--5$

As there change of sign once in f(-x) so there is one  real toot as per descartes' rule of signs

so there are 2 real roots


2022/033) Let p be a prime and r is a number less than p. Show that $\frac{(p-1)!}{r!(p-r)!}$ is an integer

Number of ways of choosing r objects from p objects is $\frac{(p!}{r!(p-r)!}$. so $\frac{(p!}{r!(p-r)!}$ is an integer

or $\frac{p(p-1)!}{r!(p-r)!}$ is integer . Now because p does not divide the denominator  because p is prime dividing by p we get the result 

Wednesday, March 30, 2022

2022/032) Show that $\sin ^3 18^\circ + \sin ^2 18^\circ = \frac{1}{8}$

we have $\sin\, 54^\circ = \cos \, 36^\circ$

let $x = 18^\circ$

so we have $\sin 3x = \cos 2x$

Or $3 \sin \, x - 4\sin ^3 x = 1 - 2 \sin ^2 x$

putting $\sin \,x = y$ we get

$3y - 4y^3 = 1 - 2y^2$

or $4y^3 - 2y^2 - 3 y + 1 = 0$

or we see that (y-1) is a factor as putting above as f(y) and find f(1) = 0 

by division we get $(y-1)(4y^2 + 2y -1) = 0$

as $\sin\,18^\circ$ is not zero so $4y^2 + 2y - 1=0\cdots(1)$

we need to show $y^3 + y^2 = \frac{1}{8}$

from (1) $4y^2 = - 2y + 1\cdots(2)$

or $8y^3 = 2y(-2y + 1) = - 4y^2 + 2y  = - -4y^2 + 1 - 4y^2$ (from (2))

or $8y^3 + 8y^2 = 1$

or $y^3 + y^2 = \frac{1}{8}$

   proved 

 

Saturday, March 26, 2022

2022/031) The number of real roots of the equation $3x^2 - 4 |x^2-1| + x-1 = 0$ is

 there are 2 cases $x^2 >= 1$

this gives $3x^2 - 4(x^2-1) + x -1 = 0$

or $-x^2 + x + 3 = 0$

or $x^2 -x  - 3 = 0$

this has two slutions one in $(1,\infty$ and another is $(-\infty, -1)$ 

for for the other case we have $3x^2 + 4(x^2-1) + x -1 = 0$ or $7x^2 + x - 5 = 0$ f(0) = -5, f(1) = 3 f(-1) = = 1 so there are 2 roots one between 0 and 1 and another between 0 and -1. 

So there are 4 solutions 

2022/030) Factor $x^4+3 x^2 y^2+2 y^4+4 x^2+5 y^2+3$

we have

 $x^4+3 x^2 y^2+2 y^4+4 x^2+5 y^2+3$

$= x^4+3 x^2 y^2+4 x^2 + 2 y^4 +5 y^2+3$ putting them in descending power of x

$=x^4+ x^2(3 y^2 +4) +  (2y^2 + 1)(y^2 + 3)$ factoring the expression involving y only

$=x^4+ x^2((2y^2 + 1) + (y^2 +3) +  (2y^2 + 1)(y^2 + 3)$ split the middle term as $3y^2 + 4 = 2y^2 + 1 + y^3 + 3$

$= x^2(x^2 + 2y^2 + 1) + (y^2 + 3)(x^2 + 2y^2  +1 ) = (x^2 +2y^2 + 1)(x^2 + y^2 + 3)$

Monday, March 21, 2022

2022/029) Show that $\frac{1}{29} + \frac{1}{31} + ... \frac{1}{55}$ when the sum is put in the form $\frac{p}{q}$ then p is divisible by 83

Using $\frac{1}{n} + \frac{1}{a-n} = \frac{a}{n(a-n)}$
We get $\frac{1}{n} =  \frac{a}{n(a-n)}-\frac{1}{a-n}$
Putting a= 83 we get
$\frac{1}{n} =  \frac{83}{n(83-n)}-\frac{1}{83-n}$

Running n from 29 upto 55 in increment of 2 we have

$\sum_{n=14}^{27} \frac{1}{2n+1}=\sum_{n=14}^{27} (\frac{83}{(2n+1)(82-2n)}-\frac{1}{82-2n})$

Or $\sum_{n=14}^{27} \frac{1}{2n+1}=83\sum_{n=14}^{27}\frac{1}{(2n+1)(82-2n)}- \sum_{n=14}^{27}\frac{1}{82-2n}$

As $\sum_{n=14}^{27}\frac{1}{(2n+1)(82-2n)}$ when we take the sum  we shall get it of the form $\frac{p}{q}$ where q does not have a divisor 83. as 83 does not divide any denominator

So  $\sum_{n=14}^{27} \frac{1}{2n+1}=\frac{83p}{q}- \sum_{n=14}^{27}\frac{1}{82-2n}\cdots(1)$

Further $82-2n = 2k => n = 41-k$

So $\sum_{n=14}^{27}\frac{1}{82-2n} = \sum_{n=14}^{27}\frac{1}{2n}\cdots(2)$

 Now the given expression

= $\sum_{n=0}^{27} \frac{1}{2n+1 } -  \sum_{n=1}^{27} \frac{1}{2n}$

= $\sum_{n=0}^{13} \frac{1}{2n+1 } -  \sum_{n=1}^{27} \frac{1}{2n} + \sum_{n=14}^{27} \frac{1}{2n+1 }  $

= $\sum_{n=0}^{13} \frac{1}{2n+1 } -  \sum_{n=1}^{27} \frac{1}{2n} + \frac{83p}{q}- \sum_{n=14}^{27}\frac{1}{82-2n}$

= $\sum_{n=0}^{13} \frac{1}{2n+1 } -  \sum_{n=1}^{27} \frac{1}{2n} + \frac{83p}{q}- \sum_{n=14}^{27}\frac{1}{2n}$

=  $\sum_{n=0}^{13} \frac{1}{2n+1 } -  \sum_{n=1}^{13} \frac{1}{2n} + \frac{83p}{q}- 2 \sum_{n=14}^{27}\frac{1}{2n}$

=  $\sum_{n=0}^{13} \frac{1}{2n+1 } -  \sum_{n=1}^{13} \frac{1}{2n} + \frac{83p}{q}-  \sum_{n=14}^{27}\frac{1}{n}$

=  $\sum_{n=0}^{13} \frac{1}{2n+1 } -  \sum_{n=1}^{13} \frac{1}{2n} + \frac{83p}{q}-   \sum_{n=7}^{13}\frac{1}{2n} -  \sum_{n=7}^{13}\frac{1}{2n+1}$

=  $(\sum_{n=0}^{13} \frac{1}{2n+1 } -  \sum_{n=7} ^{13} \frac{1}{2n+1}) + \frac{83p}{q}-   \sum_{n=1}^{6}\frac{1}{2n} -  \sum_{n=7}^{13}\frac{2}{2n}$
=  $\sum_{n=0}^{6} \frac{1}{2n+1 } + \frac{83p}{q}-   \sum_{n=1}^{6}\frac{1}{2n} -  \sum_{n=7}^{13}\frac{1}{n}$

= $\frac{83p}{q} + \sum_{n=0}^{13} \frac{1}{2n+1 } -  \sum_{n=1}^{13} \frac{1}{2n} -   \sum_{n=7}^{13}\frac{1}{2n} -  \sum_{n=7}^{13}\frac{1}{2n+1}$
= $\frac{83p}{q} + \sum_{n=0}^{13} \frac{1}{2n+1 } -  \sum_{n=7}^{13} \frac{1}{2n+1} -   \sum_{n=1}^{13}\frac{1}{2n} -  \sum_{n=7}^{13}\frac{1}{2n}$
= $\frac{83p}{q} + \sum_{n=0}^{6} \frac{1}{2n+1 } - \sum_{n=1}^{6}\frac{1}{2n} - \sum_{n=7}^{13}\frac{1}{2n}-  \sum_{n=7}^{13}\frac{1}{2n}$
= $\frac{83p}{q} + \sum_{n=0}^{6} \frac{1}{2n+1 } - \sum_{n=1}^{6}\frac{1}{2n} - \sum_{n=7}^{13}\frac{1}{n}$
= $\frac{83p}{q} + \sum_{n=0}^{6} \frac{1}{2n+1 } - \sum_{n=1}^{6}\frac{1}{2n} - \sum_{n=7}^{13}\frac{1}{n}$
= $\frac{83p}{q} + \sum_{n=0}^{6} \frac{1}{2n+1 } - \sum_{n=1}^{6}\frac{1}{2n} - \sum_{n=7}^{13}\frac{1}{n}$
= $\frac{83p}{q} + \sum_{n=0}^{6} \frac{1}{2n+1 } - \sum_{n=1}^{6}\frac{1}{2n} - (\sum_{n=3}^{6}\frac{1}{2n+1} + \sum_{n=4}^{6}\frac{1}{2n})$
= $\frac{83p}{q} + \sum_{n=0}^{2} \frac{1}{2n+1 } - \sum_{n=1}^{6}\frac{1}{2n} - \sum_{n=4}^{6}\frac{1}{2n}$
= $\frac{83p}{q} + \sum_{n=0}^{2} \frac{1}{2n+1 } - \sum_{n=1}^{3}\frac{1}{2n} - \sum_{n=4}^{6}\frac{1}{2n} -  \sum_{n=4}^{6}\frac{1}{2n}$
= $\frac{83p}{q} + \sum_{n=0}^{2} \frac{1}{2n+1 } - \sum_{n=1}^{3}\frac{1}{2n} - \sum_{n=4}^{6}\frac{1}{n} $
= $\frac{83p}{q} + 1 + \frac{1}{3} +  \frac{1}{5} - \frac{1}{2} - \frac{1}{4} - \frac{1}{6} - \frac{1}{4} - \frac{1}{5} - \frac{1}{6}$
$=\frac{83p}{q}$

Numerator is divisible by 83 and denominator is not and 83 is prime . hence proved 

Saturday, March 19, 2022

2022/028) Solve $\sin ^5 x + \cos^5 x = 1$

 We know as $-1 \le \sin\, x \le 1$ so $sin^5 x \ le sin ^2 x$. they are same when sin x = 0 or 1 otherwise  $\sin^5 x \lt sin ^2 x$

similarly $\cos^5 x \le \cos ^2 x$. they are same when cos  x = 0 or 1 otherwise  $cos^5 x \ lt cos  ^2 x$

so $\sin ^5 x + \cos^5 x \le \sin ^2 x + \cos^2 x$ or 1 they are same when $\sin \, x =$ 0 or and $\cos\,x$ = 0 or 1 that is one of them is zero and another 1

this is possible when $\sin\, x = 0$ and $\cos\,x = 1$ that is $x = 2n\pi$

or $\sin\, x = 1$ and $\cos\,x = 0$ that is $x = (2n + \frac{1}{2}) \pi$

hence $x \in \{ 2n\pi, (2n + \frac{1}{2}) \pi \} $



Wednesday, March 16, 2022

2022/027) If $\sum_{k=1}^n \frac{2k+1}{(k^2+k)^2} = .9999$ find n

 We have

$\frac{2k+1}{(k^2+k)^2} = \frac{(k+1)^2 - k^2}{(k+1)^2 k^2} =  \frac{1}{k^2} - \frac{1}{(k+1)^2}$


The above is a telescopic sum and we have

 $\sum_{k=1}^n \frac{2k+1}{(k^2+k)^2} = \frac{1}{1} - \frac{1}{(n+1)^2} = .9999$

so  $\frac{1}{(n+1)^2} = 1- .9999 = .0001 = \frac{1}{100^2}$

n+ 1 = 100 or n = 99 

Monday, February 21, 2022

2022/026) Let P(x) be any polynomial with integer coefficients such that P(21)=17,P(32)=−247and P(37)=33. Prove that if P(N)=N+51 for some integer N, then N=26.

Using the property of polynomial that $a-b | p(a)- p(b)$ we have between N and 21

$N-21 | P(N) - p(21)$

As $P(N) = N+ 51$

We get $N-21 | N+ 51 -  17$

or $N-21 | N + 34| or | N - 21 |(N+ 34) - (N-21) $

or $N-21 | 55$

This gives N-21 = 1 or 5 or 11 or 55 or -1 or -5 or - 11 or -55

So N = 26 or 32 or 36 or 76 or 20 or 16 or 10 or - 34 (1)

Now with 32 we get $N- 32 | N + 51 - (-247)$ or $N-32 | 330$ (2)

Finally with 37 we get $N -37 | N + 51 - 33$ | or $N- 37 | 55$

from  set of (1) we get 26 or 32 or 36 

from above (3) values we get   N = 26

so N  = 26 is the answer

 

2022/025) if $a-\frac{1}{a} = b$, $b-\frac{1}{b} = c$, $c-\frac{1}{c} = a$ evaluate $\frac{1}{ab} + \frac{1}{bc} + \frac{1}{ca}$

 We are given $a-\frac{1}{a} = b\cdots(1)$

$b-\frac{1}{b} = c \cdots(2)$

$c-\frac{1}{c} = a\cdots(3)$

From (1)$\frac{1}{a} = a-b $

dividing both sides by b

$\frac{1}{ab}  =  \frac{a}{b} - 1  = a(b-c)  -1 $ using |(2)

similarly

$\frac{1}{bc}  =  \frac{b}{c} - 1  = b(c-a)  -1 $

and $\frac{1}{ca} = \frac{c}{a} - 1 = -c(a-b) = 1 $

adding above 3 we have $\frac{1}{ab} + \frac{1}{bc} + \frac{1}{ca}= - 3 $


Saturday, February 19, 2022

2022/024) Given $a_1^2+5a_2^2=10,\,a_2b_1-a_1b_2=5$ and $a_1b_1+5a_2b_2=\sqrt{105}$ for $a_1,\,a_2,\,b_1,\,b_2\in R$, evaluate $b_1^2+5b_2^2$.



we have

$(p^2+q^2)(l^2 + m^2) = (pl-qm)^2+ (pm + ql)^2$




putting $p= a_1$, $q = \sqrt{5} a_2$ ,$l= b_1$, $m = \sqrt{5} b_2$



we get $(a_1^2 + 5a_2^2)(b_1^2+5b_2)^2 = (a_1b_1 + 5a_2b_2)^2+5(a_1b_2 - a_2b_1)^2$



putting values from given conditions we get



$10(b_1^2+5b_2^2) = 105 + 5 * 5^2$

or $10(b_1^2 + 5b_2^2) = 230$

or $b_1^2 + 5b_2^2 = 23$


2022/023) Solve for x $\sqrt{3-x} - \sqrt{1+x} >= \frac{1}{2}$

We first note that $3-x \ge 0$ and $1+x <=0$ as both are under square root and so we have $-1 \le x \le 3$

 Secondly we have the LHS is monotonically decreasing as x increases

so let uf have $f(x) = \sqrt{3-x} - \sqrt{1+x}$

We have the LHS is monotonically decreasing as x increases and if is defined in $[-1,3]$

now $f(-1) = \sqrt{3-(-1)} - \sqrt{1+{-1}} = 2$ and $f(3) = \sqrt{3-3} - \sqrt{1+3} = -2$

so we need to solve for  $\sqrt{3-y} - \sqrt{1+y} = \frac{1}{2}$

and then $y \le x \le -3$ 

we need to solve 

$\sqrt{3-y} - \sqrt{1+y} = \frac{1}{2}$

square both sides to get $3 - y + 1 + y - 2\sqrt{(3-y)(1+y)} = \frac{1}{4}$

or $4 - \frac{1}{4} = 2\sqrt{(3-y)(1+y)}$

square both sides to get $\frac{225}{64} = (3-y)(1+y)$

or $\frac{225}{64} = (3-y)(1+y)$

putting y = z + 1 we get $\frac{225}{64} = (2-z)(2+z) = 4 - z^2$

or $z^2 = \frac{31}{64}$

so $z = \pm \frac{\sqrt{31}}{8}$

so $y = 1 \pm \frac{\sqrt{31}}{8}$

now whether we show take the plus or minus we check  that $f(1) = \sqrt{2} - \sqrt{2} = 0$ wich does not sattisfy the condition so we must have -ve numebr and hence

$y =    1  - \frac{\sqrt{31}}{8}$

or $-1 \le x \le  1  - \frac{\sqrt{31}}{8}$


or 

Sunday, February 13, 2022

2022/022) Find all solutions in positive integers x, y, z of the equation $\frac{x}{y} + \frac{y}{z} + \frac{z}{x} = 3$

We have by AM GM inequaity among $\frac{x}{y}$ ,$\frac{y}{z}$, $\frac{z}{x}$

we have 

$\frac{\frac{x}{y} + \frac{y}{z} + \frac{z}{x}}{3} >= \sqrt[2]{\frac{x}{y} * \frac{y}{z} * \frac{z}{x}}$

 or  $\frac{\frac{x}{y} + \frac{y}{z} + \frac{z}{x}}{3} >= \sqrt[2]{1}$

or  $\frac{x}{y} + \frac{y}{z} + \frac{z}{x}>=  3$

this is greater than 3 if all are not equal and is 3 if all are equal

or  $\frac{x}{y} = \frac{y}{z} = \frac{z}{x}$

or x = y = z (any integer)  


2022/021) find rational x,y z, such that $x^2 + y^2 + y^2 + x + y + z =1

 we have 

$x^2 + y^2 + z^2 + x + y + z = 1$

or $x^2 + x + y^2 + y + z^2 + z = 1$

we shall complete the square so let us multiply 4 to avoid fraction

$(4x^2 + 4x ) + (4y^2 + 4y) + (4z^2 + 4z) = 4$

adding 1 to each term on the left

 $(4x^2 + 4x + 1) + (4y^2 + 4y + 1) + (4z^2 + 4z+ 1) = 7$

or $(2x+1)^2 + (2y +1)^2 + (2z+1)^2 = 7$

multipying by LCM of denomiantor of 2x+1,2y+1,2z+1 we get

$p^2 + q^2 + r^2 = 7m^2$ for integers p,q,r,m.

this has a least m which has a solution

let m be even say 2n

then  all a,b,c have to be even because RHS is divisible by 4 and if any one to 3 numbers is odd then working in mod 4 and summing we get a remainder of 1to 3 when divided by 4.

so all are even

so m cannot be even

so m is odd.

because m is say m = 2k+1

so $7m^2 = 7(2k+1)^2 = 7 * 4k^2 + 7 * 4k + 1$

ot $7m^2 = 7 * 4k(k+1) + 7$

which leaves a remainder 7 when divided by 8

a number if odd on squaring leaves a remainder 1 when divided by 8 and if even then remainder is 0

sum of 3 such numbers (0 or 1) cannot be 7.

so the equation has has no solution and hence original equation has no solution.

   

Saturday, February 12, 2022

2022/020) Given $m^2 = n +2 $, $m^2= n + 2$ and m and n are unequal Compute $4mn - m^3 - n^3$

we are given 

$m^2 = n +2 \cdots(1)$

$n^2 = m + 2\cdots(2)$

$4 mn - m^3 - n^3$

$= 4mn - m(m^2)  - n(n^2)$

$= 4mn - m(n+2)  - m(n+2)$ using (1) and (2)

$= 2mn - 2(m+n)$

or $4 mn - m^3 - n^3=  2mn - 2(m+n)\cdots(3)$ 

we need to commte mn and m + n

sutarcting (2) from (1)

$m^2 - n^2 = n - m$

as n-m is not zero divding by n-m we have

$m+n = -1\cdots(4)$

adding (1) and (2)

$m^2 + n^2 = (m+n) + 4 = -1 + 4 = 3\cdots(5)$ putting value of m + n from (4)

or

Hence $2mn = (m+n)^2 - (m^2+n^2) =  1- 3 = -2$

putting the values from (4) and (6) in (3) we get  

 $4 mn - m^3 - n^3=  2mn - 2(m+n) =  -2 -2 (-1) = 0$ 

 

Thursday, February 10, 2022

2022/019) Find real x and y such that $16^{x^2+y} + 16^{y^2 + x} =1$

 We have both $16^{x^2+ y}$ and $16^{y^2+x}$ positive.

So we can apply AM GM inquality gettting

$\frac{16^{x^2+y} + 16^{y^2+x}}{2}\ge (16^{x^2+y} 16^{y^2+x})^{\frac{1}{2}}$

or $16^{x^2+y} + 16^{y^2+x}\ge  2 (16^{x^2+y+ y^2+x})^{\frac{1}{2}}$

we are given LHS = 1 so 

$ 1 \ge 2 (16^{x^2+y+ y^2+x})^{\frac{1}{2}}$

or $ 1 \ge 16^{\frac{1}{4}}  (16^{x^2+y+ y^2+x})^{\frac{1}{2}}$

or $ 1 \ge  (16^{x^2+y+ y^2+x+ \frac{1}{2}})^{\frac{1}{2}}$

or $x^2 + y + y^2 + x + \frac{1}{2} <=0$ 

Now we combine line terms and complete square to get 

$(x^2 + x + \frac{1}{4}) + (y ^2 + y + \frac{1}{4}) <= 0$

or $(x+\frac{1}{2})^2 + (y + \frac{1}{2})^2 <=0$

it is sum of 2 squares so cannot be be -ve so

both terms and sum has to be zero givng $x = y = \frac{-1}{2}$

Tuesday, February 8, 2022

2022/018) if a,b,c are roots of equation $x^3-24x^2+183x -440=0$ find the area of the triangle whose sides are a,b,c.

 Note I picked the question at https://www.youtube.com/watch?v=iyX-q3D91Ck&t=474s and here provide a diiferent solution

Let $P(x)= x^3-24x^2+183x -440\cdots(1)$

as a,b,c are roots of the equation  P(x) = 0 so we have following 2 observations

$P(x) = (x-a)(x-b)(x-c) \cdots(2)$

and comparing oefficient of $x^2$ we have

$a+b+c = 24\cdots(3)$

Now area of the triangle say A = $\sqrt{s(s-a)(s-c)(s-c}$

where s is semiperimeter  of triangle 

so $s=\frac{a+b+c}{2}= \frac{24}{2}$ putting value of a+b+c from (3)

or $s= 12\cdots(4)$ 

from (1) putting s for x we get $P(s) = (s-a)(s- b)(s-c)$

So A = $\sqrt{s(s-a)(s-c)(s-c)} = \sqrt{sP(s)}$

so using (1) and above

$A^2 = sP(s) = s(s^3 - 24s^2 + 183 s - 440) $

putting the value of s = 12 from (4) we get

$A^2= 12 *(12^3- 24 * 12^2 + 183 * 12  - 440) = 12(12^3 - 2 * 12^3 + 183 * 12 - 440)$

Or $A^2 = 12(- 12^3 + 183 * 12 - 440) = 12(- 1728 + 2196 - 440) = 12 * 28 = 336$

or $A = \sqrt{336} = \sqrt{4^2 * 21} = 4 \sqrt{21}$  



Sunday, February 6, 2022

2022/017) find positive integer n such that $n^2-10n -22$ is same as product of digits of n

Let us fix the lower bound of n

now $n^2 -10n - 22 = (n^2 - 10n + 25) - (22+25)$ adding 25 for completing the square

$= (n-5)^2 - 47 > = 0$ as it has to be positive

so $(n-5)^2 >= 57$

or $(n-5) >= \sqrt{47}$ 

or $ n > 5 + \sqrt{47}$

as n is integer so n > 5 + 6$ as $6 < \sqrt{47} < 7$ so 5 + 6 is the largest value which does nt satisfy the condition

so $n >11$

now  n = 12 this gives product of digits = 2 and $n^2 -10n -22 = 12^2 - 10 * 12 - 22 = 2$ so they are same

so 12 is a sloution

for $n  > 12$ we have $22 < 2n$ and hence $n^2 - 10n - 22 > n^2 - 10n - 2n$

or $n^2 - 10n - 22 > n(n-12)$ and as $n-12$ is positive we have

$n^2 - 10n -22 >n$ for $ n > 13\cdots((1)$

We shall prove a little lemma:

For for any n the product of the digits of  n is less then n

proof:

let n be a k digit number starting with a.

the value of $n >= a* 10^{k-1}\cdots(2)$

the number that starts with a shall give the largest product when all rest (k-1) digits will be ragest that is 9

so product of digits  $ < a * 9^{k-1}\cdots(3)$

using (2) and (3) we get product of digts < the value n < value of exptession.

end of lemma

using the lemma and (1) we have for $n >12$ it is not possible for the expression . same as product of digits.

 

   

Saturday, February 5, 2022

2022/016) find integers m and n such that $2^m - 2^n = 2016$

 we have clearly $m > n$

taking $2^n$ common we have $2^n(2^{m-n} - 1) = 2016$

clearly $2^{n}$ is even and $2^{m-n}-1$ is odd 

so let us factor 2016 to be a power of 2 multiplied by odd

$2016 = 32 * 63$

so n= 5 and $63 + 1 = 64= 2^6$ which is a power of 2 

so m -n = 6 or m = 11

so solution m= 11, n = 6 


2022/015) What are the prime numbers P that make 37P+4 the square of a number?

if is a square let it be $x^2$ 

$37p + 4= x^2$

or $37p = (x^2-4)  = (x-2)(x+2)$

as p is prime so one number is 37 and another is prime or one number is 37p and another number is 1

if x -2 = 1 then x +2 is 5 so it is not 37p

difference between x -2 and x+ 2 is 4 so numbers are 37 and 41 or 37 and 33.

but 33 is not prime so p = 41  

Sunday, January 30, 2022

2022/014) How do you prove that $\frac{2}{1!} + \frac{2+4}{2!} + \frac{2+4+6}{3!}+ \cdots = 3e$

 We have $n^{th}$ term 

$= \frac{2 + 4 + \cdots 2n }{n!} = \frac{n(n+1)}{n!}$

So then given exprsssion

$\frac{2}{1!} + \frac{2+4}{2!}  + \frac{2+4+6}{3!}+ \cdots$

$=\sum_{n=1}^{\infty} \frac{n(n+1)}{n!}$ putting the valut from above 

$=\sum_{n=1}^{\infty} \frac{n(n-1) + 2n }{n!}$ writing n(n+1) s n(n-1) + 2n

$=\sum_{n=1}^{\infty} (\frac{n(n-1)}{n!} + \frac{2n }{n!})$

$=\sum_{n=1}^{\infty} \frac{n(n-1)}{n!}  + \sum_{n=1}^{\infty} \frac{2n }{n!}$

$=\sum_{n=2}^{\infty} \frac{n(n-1)}{n!}  + \sum_{n=1}^{\infty} \frac{2n }{n!}$ we can convert the lowerr limit from n = 1 to n =2 because for n- 1 we have n(n-1) = 0 which can be dropped

$=\sum_{n=2}^{\infty} \frac{1}{(n-2)!}  + 2 \sum_{n=1}^{\infty} \frac{1 }{(n-1)!}$

$=\sum_{n=0}^{\infty} \frac{1}{n!}  + 2\sum_{n=0}^{\infty} \frac{1 }{n!}$ by changing the limit

 $=3 \sum_{n=0}^{\infty} \frac{1}{n!}$

= 3e by using expansion form for e   

Saturday, January 29, 2022

2022/013) How do you prove that $\tan^{-1}\frac{1}{5}+ \tan^{-1} \frac{1}{7} + \tan^{-1} \frac{1}{3} + \tan^{-1} \frac{1}{8} = \frac{\pi}{4}$

Let us have $\tan^{-1}\frac{1}{5} = \alpha$

$\tan^{-1}\frac{1}{7} = \beta$

$\tan^{-1}\frac{1}{3} = \gamma$

$\tan^{-1}\frac{1}{8} = \delta$

we need to find $\alpha + \beta + \gamma + \delta$

Let us add 2 at a time $\alpha + \beta$ , $\gamma +\delta$ and then $\alpha + \beta + \gamma + \delta$

We have 

$\tan \alpha = \frac{1}{5}\cdots(1)$

and  $\tan \beta = \frac{1}{7}\cdots(2)$

So $\tan (\alpha + \beta) = \frac{\tan (\alpha) + \tan (\beta)}{1 - \tan (\alpha) \tan (\beta)}$

$= \frac{\frac{1}{5} + \frac{1}{7}}{1 - \frac{1}{5} \frac{1}{7}}$

$ = \frac{\frac{12}{35}}{ \frac{34}{35}} = \frac{12}{34} = \frac{6}{17}$ 

We have 

$\tan \gamma = \frac{1}{3}\cdots(3)$

and  $\tan \delta  = \frac{1}{8}\cdots(4)$

So $\tan (\gamma + \delta) = \frac{\tan (\gamma) + \tan (\delta)}{1 - \tan (\gamma) \tan (\delta)}$

$= \frac{\frac{1}{3} + \frac{1}{8}}{1 - \frac{1}{3} \frac{1}{8}}$

$ = \frac{\frac{11}{24}}{ \frac{23}{24}} = \frac{11}{23}$ 

now $\tan (\alpha + \beta + \gamma + \delta)$

$=\frac{\tan (\alpha + \beta)  + \tan (\gamma + \delta)}{1 - \tan (\alpha + beta) \tan (\gamma + \delta)}$

$= \frac{\frac{6}{17} + \frac{11}{23}}{1 - \frac{6}{17} \frac{17}{23}}$

$ = \frac{\frac{6 * 23 + 11 * 17}{17 * 23}}{1-  \frac{66}{17 * 23}}$

$ = \frac{6 * 23 + 11 * 17}{17 * 23 - 66}$

$=\frac{ 325}{325} = 1$

so $\tan (\alpha + \beta + \gamma + \delta) = \tan \frac{\pi}{4}$

but we need to show that $\alpha + \beta + \gamma + \delta = \frac{\pi}{4}$

as it could be $\frac{5\pi}{4}$

but as  $\alpha,\beta,\gamma, \delta$ each is less than $\frac{\pi}{4}$ so sum is less than $\pi$ so it is $\frac{\pi}{4}$ 

Thursday, January 27, 2022

2022/012) For integers x and y 34x = 43 y show that x + y is composite

 We have 34x + 34 y = 43 y + 34 y = 77 y

so 34(x+y) = 77 y

or $x+ y = \frac{77 * y}{34}$

34 is coprime to 77 , x+ y is integer so $\frac{y}{34}$ is integer  

$x+y = 77 * \frac{y}{34}$ has a factor 77 and 77 is composite so x + y is composite  

Sunday, January 23, 2022

2022/011) Find the rightmost digit of $\lfloor\frac{10^{20000}}{10^{100}+3}\rfloor$

To keep the calculation simple let n = 10^{100}$

So $10^{100} + 3 = n + 3$

So $10^{20000} = n^{200}$

We need to find remainder when $n^{200}$ is divided by n+ 3

Now $n^{200} = n^{200} - 3^{200} + 3^{200}$ 

$n^{200} - 3^{200}$ is divisible by n + 3 and $3^{200} = 9^{100}$ whcih is less than $n+3$

So we need to find the unit digit of quotient of $n^{200} - 3^{200}$ divided by n + 3

we have $n^{200} - 3^{200} = (n+3)\sum_{k=0}^{199}(-1)^k n^k 3^{199-k}$

Hence

$ \frac{n^{200}-3^{200}}{n+3}= \sum{k=0}^{199}(-1)^k n^k 3^{199-k}$

putting back $n= 10^{100}$

we get $ \frac{10^{20000}-3^{200}}{10^{100}+3}= \sum{k=0}^{199}(-1)^k (10^{200})^k 3^{199-k}$

All the trerms except when k = 0 have a factor 100 so we get unit digit is unit digit of $-3{199}$

now $3^4$ ends with 1 so $3^{199} = (3^4)^{49} * 3^3 $ ends with unit digit of $3^3$ or 7 so if we negate it it ends with 10 - 7 = 3

so ans 7 

Saturday, January 22, 2022

2022/010) Solve the equation $n! = n^3 - n$ for integer n

Note: I picked the question from  https://www.youtube.com/watch?v=uqaRfADGOtg and I provide my solution for the same in a diffrerent way.

 We see that $n!$ grows at a rate much faster than $n^3$ 

For solving this we can  fix the bound for n and check on the same.

as we see $6^3 = 216$ and $6! = 720$ which is greater

by chceking the values of n from 0 to 5 we get n = 5 satsfies the equation

so Ans $n= 5$

Friday, January 21, 2022

2022/009) Find the gcd(19! + 19, 20! + 19)

 We have $GCD(19! + 19, 20!+19)$ 

$= GCD( 19!+ 19 , 20(19! + 19) - (20!+ 19))$ using gcd(a,b) = gcd(a, na -b))

$= GCD(19! + 19, 20! + 20 * 19 - 20! -19)$

$= GCD(19! + 19, 20 * 19 - 19)$

$= GCD(!9! + 19, 361)$

Now $361 = 19^2$

$19! + 19 $is 1$9(18! + 1)$

As 19 is a prime as per wilson's theorem 18! + 1 is divisible by 19

So $19! + 19$ is divisible by $19^2 = 361$

so GCD = 361 

Monday, January 17, 2022

2022/008) Given $\sin^3 x + \cos ^3 x= \frac{1}{\sqrt{2}}$ find $\sin \, 2x$

 we have  $\sin^3 x + \cos ^3 x= \frac{1}{\sqrt{2}}$

letting $\sqrt{2} \sin\,x = s$ and $\sqrt{2} \cos\, x = c$

we have $s^3 + c^3 = 2 \sqrt{2} \frac{1}{\sqrt{2}}= 2$

or $s^3 + c^3 = 2\cdots(1)$

and $s^2 + c^2 = 2\cdots(2)$

we are interested to find $\sin\,2x$ or sc

squaring (1) we get $s^6 + c^6 + 2s^3c^3 = 4$

and cubing (2) we get $s^6 + c^6 + 3s^2c^2(s^2 + c^2) = 8$

or $s^6 + c^6 + 6s^2c^2 = 8$

from (3) and (4) we have letting sc = x

$2x^3 - 6x^2 + 4 = 0$

or $x^3 - 3x^2 + 2$   this gives x = 1

and deviding by x-1 for factoring we get $(x-1)(x^2 - 2x -2) = 0$

so x = 1 or $1\pm \sqrt{3}$ but $1 + \sqrt{3}$ being above 1 is not admissible

so x = 1 or $1-\sqrt{3}$

Friday, January 14, 2022

2022/007) If, $n^2−33,n^2−31$ and $n^2−29$ are prime numbers, then what is the number of possible values of n where n is an integer

 $n^2−33,n^2−31$ and $n^2−29$ are 3 consecutive even numbers or 3 consecutive odd numbers

they cannot be 3 consecutive even numbers as all cannot be prime

so they are 3 cosecutive odd numbers.

so one of them is divisible by 3.

so the number has to be 3 otherwise it is not prime.

so 3 numbers are 3 , 5, 7 and $n^2-33 = 3$ giving $n = \pm 6$ or number of values of n = 2


Saturday, January 8, 2022

2022/006) Prove that there are infinitely many positive integers n such that n(n+1) can be represented as a sum of 2 positive integers in atleast 2 ways

 If we choose n as a square say $m^2$ then 

$n(n+1) = m^2(m^2+1) = m^4 + m^2$

now n(n+1) is reperesented as sum of 2 squares that is $(m^2)^2 + m^2$

if $m^2$ can be represented as sum of 2 squares that is $p^2 + q^2$ this is possible as as (m,p,q) form a pythagorean triple then we have

$n(n+1) = (p^2+q^2)(m^2+1) = (pm+ q)^2 + (p-qm)^2$ this is another way 

if we chose $p=x^2-y^2, q = 2xy, m= x^2+y^2, n = (x^2+y^2)^2 $ then it satisfies the condition  (p,q,m) form a pythagrean triple)

Friday, January 7, 2022

2022/005) When $x^{1000}$ is divided by $x^2-4x+3$ what is the remainder?

 When $x^{1000}$ is divided by $x^2-4x + 3$ the raminder shall be a polynomial of degree 1 that is Ax + B

So $x^{1000} = (x^2 - 4x +3) P(x) + Ax + B$ where P(x) is quotient 

We need to find A and B

Now $x^2-4x + 3 = (x-1)(x-3)$

So $x^{1000} = (x-1)(x-3) P(x) + Ax + B$

Putting x = 1 we get $1= A + B\cdots(1)$

Putting x = 3 we get $3^{1000} = 3A + B\cdots(2)$

Subtracting (1) from (2) we get 2A = $3^{1000} -1$

Or $A = \frac{3^{1000}-1}{2}$

Putting in (1) we get $B= 1 - A = \frac{1-3^{1000}}{2}$

So remainder = $\frac{3^{1000}-1}{2}x + \frac{1-3^{1000}}{2}$


2022/004) A triangle with integral sides has a perimeter 8. What will be the area of a triangle?

First let us find the lengths of sides of the trinagle. Let a,b,c be sides of triangle

By triangle inequality we have as $a+b > c$ so $a+b+c > 2c$  or $8 > 2c$ or $ c< 4$

$c = 3 => a + b = 5$ giving a = 3 and b =2 or a =2 and b = 3

$c=2=> a+b=6$ giving a = b= 3

$c=1$ is not possible as it gives $a+b= 7$ give a a =3 , b= 4 invalid triangle

sides 3,3,2 say a =  b= 3 and c = 2

and s (semiperimeter) = 8/2 = 4

so if A is area $A^2 = s(s-a)(s-b)(s-c) = 4 * 1 * 1 * 2 = 8$ or Area = $2\sqrt{2}$

 


 

Tuesday, January 4, 2022

2022/003) Prove $x^2+y^2+5>xy+x+3y$

 We have

$(x-y)^2 >=0$ or $x^2 + y^2 >=2xy$

$(x-1)^2 >=0$ or $x^2 + 1 >= 2x$

$(y-3)^2 >= 0$ or $y^2 + 9 >= 6y$

Adding we get $2(x^2+y^2 + 5) >= 2(xy+x + 3y)$

Or $x^2+y^2 + 5 >= xy + x + 3y$

This is equal when x = y, y = 3, x= 1 which canot be true so  $x^2+y^2 + 5 > xy + x + 3y$

Monday, January 3, 2022

2022/002) Find the smallest number whose product of digits is 10000

 Because the product of digits is 10000 so this can be represented as product os single numbers.

Let us findone such number and then see if we canmake it smaller

$10000 = 2 ^ 4 * 5 ^ 4$

So if the number has 4 2's and 4 5's then the  product of the digits is 10000. This is an 8 digit number.  this can be mad smaller if we can multiply 2 or more one digit number out of  these and getting one digit number, multiplying 3 2's we get 8 and digits are 2, 8, 5,5,5 ,5 giviing the product 1000. the smallest number that can be formed is 255558 and this is the answer   

Sunday, January 2, 2022

2022/001) Solve in integers $x+y+z=3$ and $x^3+y^3 + z^3 = 3$

We are given

$x+y+ z = 3\cdots(1)$

$x^3 + y^3 + z^ 3 = 3\cdots(2)$

cubing (1) we get $(x+y+z)^3 = 3^3$

or $x^3 + y^3 + z^3 + 3(x+y)(y+z)(z+x) = 27$

or $3 + 3(x+y)(y+z) (z+x) = 27$ putting thw vaue of $x^3 + y^3 + z^3 = 3$ from (2)

or  $(x+y)(y+z)(z+ x) = 8$

now $((x+y) + (y+z) + (z+ x)) = 2 ( x + y + z) = 6$ usng (1)

we need to find 3 numbers whose sum is 6 and product is 8. They are (2,2,2) or (4,1,1) and other combinations fail

taking x + y = 2 , y + z = 2, z+ x = 2 and using (2) that is x + y + z = 3 we get x=1,y=1,z = 1

 taking x + y = 4 , y + z = 1, z+ x = 1 and using (2) that is x + y + z = 3 we get x=5,y=z = - 4

takinn n rotaton other combinations we get x=y = -4, z = 5 as one solution and z = z = -4 and y =5 as another solution

so $\{x,y,z\} = \{2,2,2\}$ or $\{5,-4,-4\}$ (that is any permuation of the same)